Difference between revisions of "Number Theory Problems and Results"

m (Solution 1 to Problem 1(Euler's Totient Theorem))
m
 
(11 intermediate revisions by the same user not shown)
Line 1: Line 1:
This is a page where you can learn about number theory and its applications. There are important results and practice problems.
+
This is a page that was deemed obsolete. The contents were moved to a new webpage.
  
==Introduction==
+
[[Number Theory Problems Collection]]
 
 
==Results==
 
 
 
Here includes some important results for number theory.
 
 
 
===Wilson's Theorem===
 
 
 
For a prime number <math>p</math>, we have
 
 
 
<cmath>(p-1)! \equiv -1 \pmod p</cmath>
 
 
 
 
 
'''Example:'''
 
 
 
For any prime number <math>p</math>, we have
 
 
 
<cmath>\dbinom{2p}{p} \equiv 2 \pmod p</cmath>
 
 
 
''Proof:''
 
 
 
<cmath>\dbinom{2p}{p} = \frac{(p+1) \cdot (p+2) \cdot \dots \cdot 2p}{p!}</cmath>
 
 
 
<cmath>=\frac{2 \cdot (p+1) \cdot (p+2) \cdot \dots \cdot (2p-1)}{(p-1)!}</cmath>
 
 
 
Note that by Wilson's Theorem,
 
 
 
<cmath>(p+1) \cdot (p+2) \cdot \dots \cdot (2p-1) \equiv (p-1)! \equiv -1 \pmod p</cmath>
 
 
 
, so
 
 
 
<cmath>- \dbinom{2p}{p} \equiv -2 \pmod p</cmath>
 
 
 
so the result follow. <math>\square</math>
 
 
 
===Format's Little Theorem===
 
 
 
For a prime number <math>p</math> and integer <math>a</math> that <math>p</math> does not divide, we have
 
 
 
<cmath>a^{p-1} \equiv 1 \pmod p</cmath>
 
 
 
 
 
===Euler's (Totient) Theorem===
 
 
 
For relatively prime numbers <math>m</math> and <math>a</math>, we have
 
 
 
<cmath>a^{\phi (m)} \equiv 1 \pmod m</cmath>
 
 
 
'''Example:'''
 
 
 
In [[2017 AIME I Problems/Problem 14|2017 AIME I Problem 14]], at the end, we used the Euler Totient Theorem to obtain that <math>2^{100} \equiv 1 \pmod 125</math>
 
 
 
==Problems==
 
 
 
1. Suppose
 
 
 
<cmath>x \equiv 2^4 \cdot 3^4 \cdot 7^4+2^7 \cdot 3^7 \cdot 5^6 \pmod{7!}</cmath>
 
 
 
Find the remainder when <math>\min{x}</math> is divided by 1000.
 
 
 
2. (Very hard) Let <math>\pi (n)</math> denote the number of primes that is less than or equal to <math>n</math>.
 
 
 
Show that there exist numbers <math>c_1</math> and <math>c_2</math> such that
 
 
 
<cmath>c_1 \frac{x}{\log{x}}< \pi (x) < c_2 \frac{x}{\log{x}}</cmath>
 
 
 
==Solution==
 
 
 
===Solution 1 to Problem 1(Euler's Totient Theorem)===
 
 
 
We first simplify <math>2^4 \cdot 3^4 \cdot 7^4+2^7 \cdot 3^7 \cdot 5^6:</math>
 
 
 
<cmath>2^4 \cdot 3^4 \cdot 7^4+2^7 \cdot 3^7 \cdot 5^6=42^4+6 \cdot 30^6=(\frac{5 \cdot 6 \cdot 7}{5})^{\phi (5)}+6\cdot (\frac{5 \cdot 6 \cdot 7}{7})^{\phi (7)}+0 \cdot (\frac{5 \cdot 6 \cdot 7}{6})^{\phi (6)}</cmath>
 
 
 
so
 
 
 
<cmath>x \equiv (\frac{5 \cdot 6 \cdot 7}{5})^{\phi (5)}+6\cdot (\frac{5 \cdot 6 \cdot 7}{7})^{\phi (7)}+0 \cdot (\frac{5 \cdot 6 \cdot 7}{6})^{\phi (6)} \equiv (\frac{5 \cdot 6 \cdot 7}{5})^{\phi (5)} \equiv 1 \pmod{5}</cmath>
 
 
 
<cmath>x \equiv (\frac{5 \cdot 6 \cdot 7}{5})^{\phi (5)}+6\cdot (\frac{5 \cdot 6 \cdot 7}{7})^{\phi (7)}+0 \cdot (\frac{5 \cdot 6 \cdot 7}{6})^{\phi (6)} \equiv 0 \cdot (\frac{5 \cdot 6 \cdot 7}{6})^{\phi (6)} \equiv 0 \pmod{6}</cmath>
 
 
 
<cmath>x \equiv (\frac{5 \cdot 6 \cdot 7}{5})^{\phi (5)}+6\cdot (\frac{5 \cdot 6 \cdot 7}{7})^{\phi (7)}+0 \cdot (\frac{5 \cdot 6 \cdot 7}{6})^{\phi (6)} \equiv 6 \cdot (\frac{5 \cdot 6 \cdot 7}{7})^{\phi (7)} \equiv 6 \pmod{7}</cmath>.
 
 
 
where the last step of all 3 congruences hold by the [[Euler's Totient Theorem]]. Hence,
 
 
 
<cmath>x \equiv 1 \pmod{5}</cmath>
 
 
 
<cmath>x \equiv 0 \pmod{6}</cmath>
 
 
 
<cmath>x \equiv 6 \pmod{7}</cmath>
 
 
 
Now, you can bash through solving linear congruences, but there is a smarter way. Notice that <math>5|x-6,6|x-6</math>, and <math>7|x-6</math>. Hence, <math>210|x-6</math>, so <math>x \equiv 6 \pmod{210}</math>. With this in mind, we proceed with finding <math>x \pmod{7!}</math>.
 
 
 
Notice that <math>7!=5040= \text{lcm}(144,210)</math> and that <math>x \equiv 0 \pmod{144}</math>. Therefore, we obtain the system of congruences :
 
 
 
<cmath>x \equiv 6 \pmod{210}</cmath>
 
 
 
<cmath>x \equiv 0 \pmod{144}</cmath>.
 
 
 
Solving yields <math>x \equiv 2\boxed{736} \pmod{7!}</math>, and we're done. <math>\square</math>~Ddk001
 
 
 
==See Also==
 
 
 
* [[Problem Collection]]
 

Latest revision as of 20:33, 4 January 2025

This is a page that was deemed obsolete. The contents were moved to a new webpage.

Number Theory Problems Collection